W18: Med-Surg EAQ Quiz - Respiratory System

Réussis tes devoirs et examens dès maintenant avec Quizwiz!

What is the normal value of functional residual capacity? 1 2.5 L 2 3.5 L 3 4.5 L 4 6.0 L

1, 2.5 L The normal value of functional residual capacity is 2.5 L. The normal value of inspiratory capacity is 3.5 L. The normal value of vital capacity is 4.5 L.The normal value of total lung capacity is 6.0 L.

A nurse reviews the chest examination reports of four clients with respiratory disorders. Which client's findings indicate atelectasis?

A client suffering from atelectasis may have decreased fremitus, dull percussion over the affected area, and crackle sounds upon auscultation like Client C. Decreased chest wall movements, hyperresonance, and wheezing indicate asthma in Client A. Client B with increased vibrations over the chest wall above effusion, dull percussion, and diminished or absent breath sounds over the affected area may have a pleural effusion. Client D with increased fremitus over the affected area, dull percussion over the affected area, and bronchial sounds upon auscultation may have pneumonia.

Which assessment finding is considered the earliest sign of decreased tissue oxygenation? 1. Cyanosis 2. Cool, clammy skin 3. Unexplained restlessness 4. Retraction of interspaces on inspiration

Unexplained restlessness Unexplained restlessness is considered the earliest sign of decreased oxygenation. The other assessment findings, such as cyanosis, cool, clammy skin, and retraction of interspaces on inspiration, are considered late signs of decreased oxygenation.

A registered nurse is educating a client who has just undergone thoracentesis on the manifestations of pneumothorax. Which statements made by the client indicate effective learning? Select all that apply. 1 "I'll report any instance of blue skin right away." 2 "I'll report any feeling of air hunger immediately." 3 "I'll report any decrease in heart rate immediately." 4 "I'll call you right away if my nagging cough disappears." 5 "I'll call you right away if my shallow breathing goes away."

1,2 Client teaching regarding the manifestations of pneumothorax, partial or complete collapse of the lung, which may occur in the 24 hours after thoracentesis, is important. Manifestations that require immediate contact with the nearest emergency department are cyanosis and a feeling of air hunger. Other findings that must be reported include rapid heart rate (not decreased), a new-onset nagging cough (rather than the disappearance of such a cough), and rapid and shallow respirations (not the cessation of such respirations).

A 67-year-old client has tested positive for influenza A. The client also has asthma. Which drug would the nurse recommend be avoided in this client? 1 Ribavirin 2 Zanamivir 3 Oseltamivir 4 Amantadine

2. Zanamivir Zanamivir is used with caution in clients who have asthma or chronic obstructive pulmonary disease (COPD) and in older adults. Ribavirin is used for the treatment of severe influenza B. Oseltamivir may be used in treating both influenza A and B. Amantadine may be used for the treatment of influenza A.

A client is admitted to the hospital with a diagnosis of restrictive airway disease. The nurse expects the client to exhibit which early signs of respiratory acidosis? Select all that apply. 1 Headache 2 Irritability 3 Restlessness 4 Hypertension 5 Lightheadedness

1,2,3 Headache is a symptom of cerebral hypoxia associated with early respiratory acidosis [1] [2]. Irritability is a sign of cerebral hypoxia associated with early respiratory acidosis. Restlessness is a sign of cerebral hypoxia associated with early respiratory acidosis. Hypotension, not hypertension, is a key feature of acidosis. Lightheadedness is a symptom of respiratory alkalosis, not acidosis.

A client has a left pneumonectomy. Which nursing intervention is critical when the client regains consciousness in the postanesthesia care unit? 1 Assessing for pain 2 Removing the airway 3 Encouraging deep breathing 4 Positioning on the right side

3. Encouraging deep breathing Encouraging deep breathing helps to maintain airway patency and prevent atelectasis by raising intrapleural pressure. Although important, pain is not life threatening. The airway should be removed only after the gag reflex returns. Positioning on the right side restricts right lung expansion; if the sutured bronchus opens, fluid may drain from the operative area into the remaining lung.

When caring for an intubated client receiving mechanical ventilation, the nurse hears the high-pressure alarm. Which action is most appropriate? 1 Remove secretions by suctioning. 2 Lower the setting of the tidal volume. 3 Check that tubing connections are secure. 4 Obtain a specimen for arterial blood gases (ABGs).

1. Remove secretions by suctioning. Secretions in the airway will increase pressure by blocking air flow and must be removed. The nurse must identify/correct the problem so that the set tidal volume can be delivered. Connections that are not intact would cause a low-pressure alarm. ABGs are used to assess client status, but they are not taken each time a pressure alarm is heard. Test-Taking Tip: Calm yourself by closing your eyes, putting down your pencil (or computer mouse), and relaxing. Deep-breathe for a few minutes (or as needed, if you feel especially tense) to relax your body and to relieve tension.

A nurse is caring for several clients in the intensive care unit. Which is the greatest risk factor for a client to develop acute respiratory distress syndrome (ARDS)? 1 Aspirating gastric contents 2 Getting an opioid overdose 3 Experiencing an anaphylactic reaction 4 Receiving multiple blood transfusions

1. Aspirating gastric contents Aspirating gastric contents is a common cause of ARDS. Gastric enzymes injure alveolar-capillary membranes, which release inflammatory mediators; the process progresses to pulmonary edema, vascular narrowing and obstruction, pulmonary hypertension, and impaired gas exchange. Getting an opioid overdose is not as common a cause of ARDS as is aspiration pneumonia; this more likely will cause depressed respirations. Although anaphylaxis may cause ARDS, it is not a common cause. Although multiple blood transfusions have been known to precipitate ARDS, they are not a common cause. Test-Taking Tip: As you answer each question, write a few words about why you think that answer is correct; in other words, justify why you selected that answer. If an answer you provide is a guess, mark the question to identify it. This will permit you to recognize areas that need further review. It will also help you to see how correct your "guessing" can be. Remember: on the licensure examination you must answer each question before moving on to the next question.

Which surgical procedure is appropriate for the removal of a vocal cord due to laryngeal cancer? 1 Cordectomy 2 Tracheotomy 3 Total laryngectomy 4 Oropharyngeal resections

1. Cordectomy A cordectomy is a surgical procedure performed in clients with laryngeal cancer; this surgery involves the removal of a vocal cord. A tracheotomy is a surgical incision in the trachea for the purpose of establishing an airway. A total laryngectomy is a surgical procedure in which the entire larynx, hyoid bone, strap muscles, and one or two tracheal rings are removed. A nodal neck dissection is also done in a total laryngectomy if the nodes are involved. An oropharyngeal resection is a surgical procedure performed to treat cancer of the oropharynx.

A client presents to the emergency room with coughing and sudden wheezing. The nurse notes the client is progressing quickly into respiratory distress. The nurse identifies that the client is experiencing what problem? 1 An acute asthma attack 2 Acute bronchitis 3 Left-sided heart failure 4 Cor pulmonale

1, acute asthma Symptoms for an acute asthma attack often are wheezing, coughing, dyspnea, and chest tightness. Cough, fever, and fatigue are often symptoms exhibited with acute bronchitis. Fatigue, breathlessness, weakness, shortness of breath, and fluid accumulation in the lungs are often signs of left-sided heart failure. Tiring easily, shortness of breath with exertion, lower leg edema, chest pain, and heart palpitations often are exhibited with cor pulmonale.

What points should be considered when a client with a respiratory disorder undergoes a spiral-computed tomography (CT) scan to diagnose a pulmonary embolism? Select all that apply. 1 The test involves the administration of a contrast medium. 2 Clients should have their hydration levels assessed. 3 Clients are instructed to lie still on a hard table. 4 Clients are served shellfish before the test. 5 A client's serum creatinine level is evaluated after the test.

1,2,3 A contrast medium may be given intravenously when performing a spiral-computed tomography (CT). The nurse should make sure that the client is well hydrated before and after the procedure to help flush out the contrast medium. The nurse should instruct the client to lie still on the hard table and that the scanner will revolve around the body with clicking noises. The nurse should assess if the client is allergic to shellfish because the contrast medium used is iodine-based. The nurse should evaluate the client's blood urea nitrogen and serum creatinine before the test to assess renal function.

The primary healthcare provider is preparing to instill medication into the pleural space via thoracentesis. Which interventions does the nurse consider to be appropriate when performing a thoracentesis? Select all that apply. 1 Verify breath sounds. 2 Encourage deep breaths. 3 Observe for signs of pneumonia. 4 Ensure a chest x-ray is performed after the procedure. 5 Instruct the client to cough during the procedure.

1,2,4 Breath sounds should be verified in all lung fields after thoracentesis to rule out lung collapse. The client is encouraged to perform deep breaths to help expand the lungs. A chest x-ray should be obtained after the procedure to check for pneumothorax. Observing for signs of hypoxia and a pneumothorax is essential, but the signs of pneumonia may not be useful after thoracentesis. The client should be instructed not to talk or cough during the procedure because it may cause injury to the lungs.

A nurse is caring for a client in postoperative recovery who just had a central venous catheter inserted. The client begins to complain of chest pain. Upon further assessment, the nurse notes that the client has decreased breath sounds on the affected side. Which action should the nurse do first? 1 Administer oxygen as prescribed. 2 Notify the healthcare provider. 3 Assist with insertion of chest tube. 4 Continue to assess client's respiratory status

1. Administer oxygen as prescribed The client most likely is experiencing a pneumothorax, which is a collection of air in the pleural space. This can be caused during the insertion of a central venous catheter. During insertion, the pleural covering of the lung can be punctured by the introducer on insertion of a direct subclavian approach. Signs and symptoms of a pneumothorax include chest pain, dyspnea, apprehension, cyanosis, decreased breath sounds on the affected side, and abnormal chest x-ray findings. The nurse should first think about the "ABC's" (airway, breathing, circulation) and therefore should administer oxygen as prescribed, then notify the healthcare provider, continue to assess the client's respiratory status, and then assist with chest tube insertion if indicated.

A person's bathrobe ignites while the individual is cooking in the kitchen on a gas stove. What is the priority intervention after the flames are extinguished? 1 Assess the person's breathing. 2 Offer the person sips of water. 3 Cover the person with a warm blanket. 4 Calculate the extent of the person's burns.

1. Assess the person's breathing. A patent airway is most vital; if the person is not breathing, cardiopulmonary resuscitation (CPR) should be initiated. The person should be kept nothing by mouth because extensive burns decrease intestinal peristalsis, and the person may vomit and aspirate. Covering the person with a warm blanket is not done until the assessment for breathing is completed. Calculating the extent of the person's burns is not the priority; this assessment is done after transfer to a medical facility. Test-Taking Tip: Look for options that are similar in nature. If all are correct, either the question is poor or all options are incorrect, the latter of which is more likely. Example: If the answer you are seeking is directed to a specific treatment and all options but one deal with signs and symptoms, you would be correct in choosing the treatment-specific option.

A client enters the emergency department reporting shortness of breath and epigastric distress. What should be the triage nurse's first intervention? 1 Assess vital signs. 2 Insert a saline lock. 3 Place client on oxygen. 4 Draw blood for troponins.

1. Assess vital signs Assessment is the first step of the nursing process, and vital signs provide vital information about the client's cardiopulmonary status. Although inserting a saline lock may be done, it is not the priority. Although placing the client on oxygen may be done, it is not the priority. Administration of oxygen may alter the client's baseline vital sign results. Although drawing blood for troponins may be done, it is not the priority. Test-Taking Tip: As you answer each question, write a few words about why you think that answer is correct; in other words, justify why you selected that answer. If an answer you provide is a guess, mark the question to identify it. This will permit you to recognize areas that need further review. It will also help you to see how correct your "guessing" can be. Remember: on the licensure examination you must answer each question before moving on to the next question.

A nurse is suctioning a client's tracheostomy. Place the nursing actions in order of priority when performing this procedure.

1. Auscultate the lungs and check the heart rate 2. Turn suction on to between 80 and 120 mm Hg pressure 3. Hyperoxygenate using 100% oxygen 4. Don sterile gloves 5. Guide the catheter into the tracheostomy tube using a sterile gloved hand The status of the client should be ascertained as a baseline before starting the procedure. The suction should be turned on to check its adequacy before beginning. Because oxygen will be lost during suctioning [1] [2], the client should be oxygenated using 100% oxygen before initiating the procedure. Then the nurse should don sterile gloves to protect the client from infection and guide the catheter into the tracheostomy tube without using negative pressure.

A client's respiratory status may be affected after abdominal surgery. The nurse documents the behavioral objective for this client. What statement is a behavioral objective? 1 Demonstrates the technique of coughing and deep breathing. 2 Respirations will improve with coughing and deep breathing. 3 Coughing and deep breathing will facilitate output of secretions. 4 Will cough and deep breathe five or six times every hour while awake.

1. Demonstrates the technique of coughing and deep breathing Demonstrating the technique of coughing and deep breathing is an objective that includes observable client behavior, which is specified by amount and time and therefore is measurable. The statement that respirations will improve with coughing and deep breathing is an objective not stated in measurable terms. The statement coughing and deep breathing will facilitate output of secretions is not stated in measurable terms. Telling the client to cough and deep breathe five or six times every hour while awake is a statement, not an objective. STUDY TIP: Becoming a nursing student automatically increases stress levels because of the complexity of the information to be learned and applied and because of new constraints on time. One way to decrease stress associated with school is to become very organized so that assignment deadlines or tests do not come as sudden surprises. By following a consistent plan for studying and completing assignments, you can stay on top of requirements and thereby prevent added stress.

A client is admitted to the emergency department with dyspnea, a productive cough, and fever. The healthcare provider suspects pneumonia and writes prescriptions. Place the nursing actions in the order they should be performed.

1. Elevate the head of the bed. 2. Insert a catheter to establish venous access. 3. Obtain a sputum culture. 4. Administer the prescribed intravenous piggyback antibiotic. 5. Review the results of the sensitivity test. Breathing is the priority. Elevating the head of the bed will increase lung expansion to increase the surface area for gas exchange and help relieve dyspnea. Venous access provides a route for the administration of emergency parenteral medications if needed. A specimen for a sputum culture should be obtained before initiating antimicrobial therapy to ensure accurate results. Once a specimen for a culture is obtained, the antibiotic should be administered to treat the infectious process. The results of a culture and sensitivity test will not be available immediately because of the time needed for the organisms to proliferate. Once available, the nurse should validate that the organisms are sensitive to the prescribed antibiotic.

What is the primary responsibility of a nurse when caring for a client with a chest tube attached to a three-chamber underwater-seal drainage system? 1 Ensure maintenance of the closed system 2 Maintain mechanical suction to the system 3 Encourage the client to deep breathe and cough 4 Keep the client in the dorsal recumbent position

1. Ensure maintenance of the closed system An airtight system is needed to reestablish negative pressure and to reinflate the lung. Drainage can be maintained without mechanical suction. Encouraging coughing and deep breathing is important, but not the priority. Any position is acceptable as long as the tube is not compressed or pulled.

After a spontaneous pneumothorax, the client becomes extremely drowsy, and the pulse and respirations increase. What do these client responses indicate to the nurse? 1 Hypercapnia 2 Hypokalemia 3 Increased PO2 4 Respiratory alkalosis

1. Hypercapnia Pneumothorax results in decreased surface area for gas exchange. If unaffected pleural regions cannot compensate, carbon dioxide builds up in the blood (hypercapnia). The client will become drowsy and may lose consciousness. The body attempts to compensate by increasing respiratory and pulse rates, and by the kidneys retaining bicarbonate. Hypokalemia causes extreme muscle weakness, abdominal distention, and changes in the ECG pattern. The PO2 is decreased with a pneumothorax because of the decreased surface area for gas exchange. Respiratory acidosis occurs with an elevated PCO2.

Which condition can cause a client's partial pressure of end-tidal carbon dioxide (PETCO2) to be 50 mmHg? 1 Hypoventilation 2 Tracheal extubation 3 Pulmonary embolism 4 Total airway obstruction

1. Hypoventilation Normal PETCO2 ranges from 20 to 40mmHg. Therefore a value of 50 mmHg is considered high and may be the result of hypoventilation. Tracheal extubation, pulmonary embolism, and total airway obstruction are all possible causes of decreased PETCO2 values.

A client on a ventilator is exhibiting signs of poor oxygenation. The nurse is assessing the client for which signs? 1 Increased restlessness 2 No secretions when client is suctioned 3 PaO2 of 93 4 Skin warm and dry

1. Increased restlessness Signs of poor oxygenation in the client on a ventilator may include, but are not limited to, the following: cyanosis; PaO2 less than 90; increased restlessness or agitation; skin pale, cool, and clammy; and thick, tenacious secretions present when suctioned.

A client who had thoracic surgery complains of pain at the incision site when coughing and deep breathing. What action should the nurse take? 1 Instruct the client to splint the wound with a pillow when coughing. 2 Place the client in the supine position and inspect the site of the incision. 3 Assess the intensity of the pain and administer the prescribed analgesic. 4 Call the healthcare provider immediately and then check for wound dehiscence.

1. Instruct the client to splint the wound with a pillow when coughing. Supporting the wound with a pillow when coughing relieves some of the pain because it provides support to the incised chest wall. Pain at the incision site when coughing and deep breathing is expected; it does not indicate a need to place the client in the supine position and to inspect the wound site. Analgesics will not relieve the discomfort associated with coughing unless stress placed on the incision by coughing is relieved. Pain at the incision site just when coughing and deep breathing is expected; it does not indicate a need to call the healthcare provider and then check for wound dehiscence. Test-Taking Tip: Identifying content and what is being asked about that content is critical to your choosing the correct response. Be alert for words in the stem of the item that are the same or similar in nature to those in one or two of the options. Example: If the item relates to and identifies stroke rehabilitation as its focus and only one of the options contains the word stroke in relation to rehabilitation, you are safe in identifying this choice as the correct response.

A nurse is caring for a client with chronic obstructive pulmonary disease (COPD). Which clinical finding supports the nurse's suspicion that the client is developing cor pulmonale? 1 Peripheral edema 2 Productive coughing 3 Twitching of the extremities 4 Lethargy progressing to coma

1. Peripheral edema Cor pulmonale [1] [2] is right ventricular failure caused by pulmonary congestion; edema results from increasing venous pressure. A productive cough is symptomatic of the original condition, COPD. Although twitching of the extremities and lethargy progressing to coma may be caused by alterations in oxygen and hydrogen ion levels and their effects on the central nervous system, it is the sign of peripheral edema that directly indicates increasing venous pressure secondary to cor pulmonale. Test-Taking Tip: Stay away from other nervous students before the test. Stop reviewing at least 30 minutes before the test. Take a walk, go to the library and read a magazine, listen to music, or do something else that is relaxing. Go to the test room a few minutes before class time so that you are not rushed in settling down in your seat. Tune out what others are saying. Crowd tension is contagious, so stay away from it.

A nurse is teaching breathing exercises to a client with emphysema. What is the reason the nurse should include in the teaching as to why these exercises are necessary to promote effective use of the diaphragm? 1 The residual capacity of the lungs has been increased. 2 Inspiration has been markedly prolonged and difficult. 3 The client has an increase in the vital capacity of the lungs. 4 Abdominal breathing is an effective compensatory mechanism and is spontaneously initiated.

1. The residual capacity of the lungs has been increased. Loss of elasticity causes difficult exhalation, with subsequent air trapping. Clients who have emphysema are taught to use accessory abdominal muscles and to breathe out through pursed lips to help keep the air passages open until exhalation is complete. Expiration is difficult because of air trapping and poor elasticity. There will be decreased vital capacity. Diaphragmatic breathing is a learned mechanism that is beneficial.

A client is admitted to the hospital for cancer of the larynx, and a laryngectomy is scheduled. What should the nurse include in the postoperative teaching plan? 1 Importance of cleanliness around the site of the stoma 2 Necessity of covering the tube opening while swimming 3 Establishment of a regular schedule for suctioning the tube 4 Usage of sterile technique when caring for the tracheostomy tube

1. importance of cleanliness around the site of the stoma The procedure should be explained so the client understands that the tracheostomy can serve as an entrance for bacteria and that cleanliness is imperative. Clients with a laryngectomy may no longer swim because water will flood the lungs. Suctioning must be performed only as needed; a pattern is not necessary. Sterile technique is not required; medical aseptic technique is adequate and realistic.

What are the uses of pulmonary function tests (PFT)? Select all that apply. 1 Pulmonary function tests (PFT) can detect lung cancer. 2 Pulmonary function tests (PFT) can measure lung volume. 3 Pulmonary function tests (PFT) can assess responses to bronchodilators. 4 Pulmonary function tests (PFT) are performed on the pulmonary nodules. 5 Pulmonary function tests (PFT) can diagnose pulmonary disease.

2,3,5 Pulmonary function tests (PFT) can help assess lung volume, evaluate the responses to bronchodilators, and detect pulmonary disease. A mediastinoscopy is used to detect lung cancer. A positron emission tomography (PET) scan is used to inspect pulmonary nodules.

A client is to continue oxygen therapy at home when discharged. Which client statement indicates the need for further instruction by the nurse? 1 "I will use only grounded electrical equipment." 2 "I have a new woolen blanket to keep me warm." 3 "I have told my family they cannot smoke in the house." 4 "I will keep a pitcher of water near me so I drink enough."

2. "I have a new woolen blanket to keep me warm." An open flame or a spark from static electricity (generated by such items as leather-soled shoes; wool, silk, and nylon blankets; or ungrounded electrical appliances) can initiate an explosion and fire in the presence of higher-than-environmental oxygen levels. Grounded electrical equipment helps prevent sparks. When combined with oxygen, heat from lit cigarettes can ignite flammable material. Oxygen is drying; increased fluid intake is advisable.

A client has a persistent productive cough that becomes blood tinged. A needle biopsy is scheduled. The client tells the nurse, "During the procedure, a needle will be inserted into my back to collapse my lung." Which is the most appropriate response by the nurse? 1 "I will ask the primary healthcare provider to clarify the diagnostic procedure." 2 "Tell me more about the conversation you had with your healthcare provider." 3 "The procedure will be fast so that you will experience minimal discomfort." 4 "Your perception of the diagnostic test is incorrect."

2. "Tell me more about the conversation you had with your healthcare provider." The response "Tell me more about the conversation you had with your healthcare provider" is the best response. Exploration and collection of data are important parts of the therapeutic process; anxiety, fear, and depression can influence understanding of the procedure. Instructing the client to ask the healthcare provider to clarify the procedure is not the priority; at this point, the nurse should collect more data and then may have to refer. The response "The procedure will be fast so that you will experience minimal discomfort" is false reassurance. The response "Your perception of the diagnostic test is incorrect" will put the client on the defensive. Test-Taking Tip: The night before the examination you may wish to review some key concepts that you believe need additional time, but then relax and get a good night's sleep. Remember to set your alarm, allowing yourself plenty of time to dress comfortably (preferably in layers, depending on the weather), have a good breakfast, and arrive at the testing site at least 15 to 30 minutes early.

A client with chronic obstructive pulmonary disease is admitted to the hospital with a tentative diagnosis of pleuritis. When caring for this client, what should the nurse do? 1 Administer opioids frequently 2 Assess for signs of pneumonia 3 Give medication to suppress coughing 4 Limit fluid intake to prevent pulmonary edema

2. Assess for signs of pneumonia Clients with pleuritic disease are prone to developing pneumonia because of impaired lung expansion, air exchange, and drainage. Opioids are contraindicated because opioids depress respirations. Coughing should not be suppressed; it enhances lung expansion, air exchange, and lung drainage. Oral fluids should be encouraged; pulmonary edema does not develop unless the client has severe cardiovascular disease.

The nurse is monitoring a client who is receiving peritoneal dialysis. After the dialysate has infused, the client reports severe respiratory difficulty. Which immediate action should the nurse take? 1 Weigh the client 2 Auscultate breath sounds 3 Obtain arterial blood gases 4 Turn the client on the right side

2. Auscultate breath sounds Lung sounds should be auscultated for signs of fluid overload. Weighing the client will not correct the problem. The pulse oximetry reading and other vital signs may be obtained after the solution is drained and diaphragmatic pressure is decreased; arterial blood gases are not warranted at this time. It is not necessary to turn the client to the side. STUDY TIP: Establish your study priorities and the goals by which to achieve these priorities. Write them out and review the goals during each of your study periods to ensure focused preparation efforts.

Thick mucous gland secretions, elevated sweat electrolytes, meconium ileus, and difficulty maintaining and gaining weight are associated with which autosomal recessive disorder? 1 Cerebral palsy 2 Cystic fibrosis 3 Muscular dystrophy 4 Multiple sclerosis

2. Cystic fibrosis The early symptom of cystic fibrosis is meconium ileus, which is impacted stool in the newborn. Thick mucous secretions, salty sweat, and difficulty gaining weight because of high caloric demands are characteristics of the condition. Cerebral palsy is a motor disorder caused by damage to the brain. Muscular dystrophy is a muscular disorder. Multiple sclerosis is a condition with progressive disintegration of the myelin sheath.

A nurse is teaching a client about the use of a metered-dose inhaler with a spacer. Which statement made by the client indicates the need for further teaching? 1 "I will wait for at least 1 minute between puffs." 2 "I will shake the whole unit vigorously one or two times." 3 "I will hold my breath for at least 10 seconds after removing the mouthpiece." 4 "I will insert the mouthpiece of the inhaler into the nonmouthpiece end of the spacer."

2. I will shake the whole unit vigorously one or two times." The metered-dose inhaler should be shaken vigorously for a minimum of three or four times for proper mixing of the content inside the inhaler. A minimum of a 1-minute gap should be given in between the puffs to ensure proper movement of the medications into the lungs. After removing the mouthpiece, the client should hold his/her breath for at least 10 seconds so that the drug does not escape with exhalation. Inserting the mouthpiece of the inhaler into the nonmouthpiece end of the spacer is the correct way of closing the inhaler.

A nurse teaches a client with a diagnosis of emphysema about the importance of preventing infections. What information is most significant to include? 1. Purpose of bronchodilators 2. Importance of meticulous oral hygiene 3. Technique used in pursed-lip breathing 4. Methods used to maintain a dust-free environment

2. Importance of meticulous oral hygiene Microorganisms in the mouth are transferred easily to the tracheobronchial tree and are a source of potential infection; meticulous oral hygiene is essential to reduce the risk of respiratory infection. Bronchodilators will not prevent infection; they dilate the bronchi. Pursed-lip breathing will not prevent infection; it promotes gas exchange in the alveoli and facilitates more effective exhalation. It is impossible to maintain a dust-free environment. Test-Taking Tip: Attempt to select the answer that is most complete and includes the other answers within it. For example, a stem might read, "A child's intelligence is influenced by what?" and three options might be genetic inheritance, environmental factors, and past experiences. The fourth option might be multiple factors, which is a more inclusive choice and therefore the correct answer.

While assessing a client, the nurse finds increased vibrations over the chest wall. What condition can be inferred from this finding? 1 Atelectasis 2 Pneumonia 3 Orthopnea 4 Pneumothorax

2. Pneumonia A chest wall vibration produced via vocalization is called fremitus. During an assessment if increased vibrations are observed, the lung may be filled with fluid. This condition may signify pneumonia, lung tumors, or thick bronchial secretions. Atelectasis is due to a collapse of alveoli; this condition is characterized by reduced vibrations. Orthopnea is a condition that causes a shortness of breath when lying down. Pneumothorax is the presence of air in the sacs around the lungs, which in turn reduces the vibrations in fremitus.

Following a laryngectomy a client experiences frequent coughing episodes and copious production of secretions. How should the nurse explain this to the client? 1 The irritation of the stoma as a result of the tracheostomy tube that is in place 2 The reaction of the mucous membranes to air that is dry and cool 3 An upper respiratory inflammation caused by allergies 4 An insufficient coughing and deep breathing regimen

2. The reaction of the mucous membranes to air that is dry and cool Air is moisturized and warmed as it passes through the nasopharynx. With a laryngectomy this area is bypassed, and the tracheobronchial tree compensates by producing copious amounts of secretions. Irritation of the stoma by the tracheostomy tube will produce local irritation and a local response. Upper respiratory inflammation because of allergies is not a response to allergies but to the stress of the air that is entering the tracheobronchial tract. The air is no longer warmed or humidified by passing through the nose. Insufficient coughing and deep breathing do not create a response of coughing. Test-Taking Tip: Look for options that are similar in nature. If all are correct, either the question is poor or all options are incorrect, the latter of which is more likely. Example: If the answer you are seeking is directed to a specific treatment and all but one option deal with signs and symptoms, you would be correct in choosing the treatment-specific option.

What information should the nurse include in a discharge teaching plan for a client who recently had a laryngectomy? 1 Limit the daily intake of fluids. 2 Keep the stoma covered with a scarf. 3 Only humidified air should be breathed. 4 Mucus plugs can be removed with cotton-tipped swabs

2. keep the stoma covered with a scarf A stomal cover or scarf allows air to move into and out of the trachea but prevents particles of dirt and insects from entering the stoma. Fluids should not be limited; adequate fluids help to liquefy respiratory secretions. Humidified air is not necessary because maintenance of hydration keeps secretions liquefied and mobile, so they can be expelled. Cotton-tipped swabs should not be used because cotton threads may be inhaled. Test-Taking Tip: Try putting questions and answers in your own words to test your understanding.

In which positions should the nurse place a client who has just had a right pneumonectomy? 1 Right or left side-lying 2 High-Fowler or supine 3 Supine or right side-lying 4 Left side-lying or low-Fowler

3, supin or R side lying Supine or right side-lying permits ventilation of the remaining lung and prevents fluid from draining into the sutured bronchial stump. Lying on the unoperative side restricts left lung excursion and may allow fluid to drain into the right bronchial stump. Although the high-Fowler position promotes ventilation, it may be tiring for a postoperative client. Lying on the unoperative side restricts left lung excursion and may allow fluid to drain into the right bronchial stump.

The nurse reviews teaching with a client with cancer of the larynx who is scheduled for a total laryngectomy and radical neck dissection. The nurse concludes that the teaching is effective when the client makes which statement about what he will be able to do after surgery? 1 "After surgery, I will still be able to blow my nose." 2 "After surgery, I will still be able to sip through a straw." 3 "After surgery, I will still be able to chew and swallow food." 4 "After surgery, I will still be able to smell and differentiate odors."

3. "After surgery, I will still be able to chew and swallow food." There is still a pathway from the mouth to the stomach; eating patterns are not lost when a laryngectomy is performed. Air passes through a tracheal stoma that bypasses the nose and olfactory organs. There is no passage of air from the lungs to the nose; air is expelled through a tracheal stoma.

A nurse teaches a client with a nose fracture about routine care after rhinoplasty surgery. Which statement of the client indicates the need for further teaching? 1 "I should not sniff upwards or blow my nose." 2 "I should take aspirin if I experience bleeding." 3 "I should move slowly and remain in the semi-Fowler's position whenever possible." 4 "I should not cough forcefully or strain during bowel movements."

3. "I should take aspirin if I experience bleeding." After a rhinoplasty, aspirin and other NSAIDs are avoided in order to prevent bleeding. Sniffing upwards or blowing the nose may cause nasal strain and lead to complications. Moving slowly and remaining in the semi-Fowler's position may not cause stress on the nose. Forceful coughing and straining during bowel movements may lead to nasal bleeding.

Before signing a consent form for a total laryngectomy, a client asks, "Because part of my throat will be taken out and I will breathe through a hole in my neck, will I be able to talk like I did before I had the surgery?" Which is the nurse's best response? 1 "There are many clients who have had this operation. You'll talk again." 2 "That's a good question. I'll have the healthcare provider talk with you." 3 "You seem very concerned. Tell me what you know about your surgery." 4 "Not like before but there is nothing to worry about. We do a lot of these surgeries."

3. "You seem very concerned. Tell me what you know about your surgery." he nurse should strive to clarify misconceptions and fears before a client signs a consent form; this response promotes further communication and begins where the client is. The fact that others have had the surgery provides little solace; the remainder of the response is false reassurance and does not truthfully answer the client's question. The response "That's a good question. I'll have the healthcare provider talk with you" avoids assuming the responsibility of answering the client's question; the client needs an immediate clarification. The response "Not like before but there is nothing to worry about" denies the client's feelings and cuts off communication. Test-Taking Tip: Be aware that information from previously asked questions may help you respond to other examination questions.

A client with a suspected pulmonary embolism is scheduled for a spiral computed tomography scan. Which intervention should the nurse perform when preparing the client for the test? 1 Check the client's blood glucose levels. 2 Obtain informed consent from the client. 3 Assess if the client is allergic to shellfish. 4 Instruct the client to remove his or her dentures.

3. Assess if the client is allergic to shellfish. A spiral computed tomography scan may be used to diagnose a pulmonary embolism. Before preparing the client for the test, the nurse should assess if the client is allergic to shellfish since the contrast used in the test is iodine based. The client may be asked to remove his or her dentures while preparing for magnetic resonance imaging. An informed consent may not be needed for the spiral computed tomography; it may be required for endoscopic procedures such as a bronchoscopy. High blood glucose levels may interfere with the positron emission tomography scan; therefore, the nurse should check the blood glucose levels of the client before preparing for this test. Test-Taking Tip: Make certain that the answer you select is reasonable and obtainable under ordinary circumstances and that the action can be carried out in the given situation.

What finding would be consistent with long-standing hypoxemia in a client who reports shortness of breath? 1 Scoliosis 2 Kyphosis 3 Clubbing 4 Kyphoscoliosis

3. Clubbing Clubbing, a sign of long-standing hypoxemia, involves an increase in the angle between the base of the nail and the fingernail to 180 degrees or more. It is usually accompanied by an increase in the depth, bulk, and sponginess of the end of the finger. Scoliosis, kyphosis, and kyphoscoliosis are the spinal curvatures that affect breathing.

On admission to the intensive care unit, a client is diagnosed with compensated metabolic acidosis. During the assessment, what is the nurse most likely to identify? 1 Muscle twitching 2 Mental instability 3 Deep and rapid respirations 4 Tachycardia and cardiac dysrhythmias

3. Deep and rapid respirations Deep, rapid respirations are an adaptation to a decreased serum pH. Carbonic acid dissociates in the lungs to hydrogen ions and carbon dioxide, which helps increase the serum pH. Muscle twitching results from low serum calcium (hypocalcemia), not compensated metabolic acidosis. Mental confusion does not occur in compensated acidosis; confusion can occur in uncompensated metabolic acidosis. Tachycardia and cardiac dysrhythmias are associated with hyperthyroidism, not compensated metabolic acidosis. STUDY TIP: Answer every question. A question without an answer is the same as a wrong answer. Go ahead and guess. You have studied for the test and you know the material well. You are not making a random guess based on no information. You are guessing based on what you have learned and your best assessment of the question.

A client with pneumonia now requires use of a nonrebreathing mask to maintain adequate oxygen saturation levels. How does the nurse interpret this information? 1 The client's pneumonia is continually improving. 2 Oxygen concentrations up to 44% can be obtained. 3 Mechanical ventilation may be required next. 4 Nasal cannula may be used while the client is eating

3. Mechanical ventilation may be required next. A nonrebreathing mask is used when the client requires higher oxygen concentrations and the condition is worsening. If the nonrebreathing mask does not improve oxygen saturation, the next steps to improving gas exchange and oxygenation are intubation and mechanical ventilation. Oxygen concentrations up to 90% can be achieved. Nasal cannula would not be advised, as the client requires more oxygen than can be delivered through this method.

The nurse is reviewing the client's health history. With which diagnosis is a client most likely to exhibit hemoptysis? 1 Anemia 2 Pneumonia 3 Tuberculosis 4 Leukocytosis

3. Tuberculosis Hemoptysis is expectoration of blood-stained sputum derived from the lungs, bronchi, or trachea; this is a clinical manifestation of tissue erosion caused by tuberculosis. Anemia does not cause bleeding, but it may be caused by bleeding. Pneumonia causes sputum as a result of inflammation, but the sputum usually is yellow, not bloody. Leukocytosis is increased white blood cells; it does not cause hemoptysis.

A client reports left-sided chest pain after playing racquetball. The client is hospitalized and diagnosed with left pneumothorax. When assessing the client's left chest area, the nurse expects to identify which finding? 1 Dull sound on percussion 2 Vocal fremitus on palpation 3 Rales with rhonchi on auscultation 4 Absence of breath sounds on auscultation

4. Absence of breath sounds on auscultation The left lung is collapsed; therefore, there are no breath sounds. A tympanic, not a dull, sound will be heard with a pneumothorax [1] [2]. There is no vocal fremitus because there is no airflow into the left lung as a result of the pneumothorax. Rales with rhonchi will not be heard because there is no airflow into the left lung as a result of the pneumothorax. Test-Taking Tip: Practicing a few relaxation techniques may prove helpful on the day of an examination. Relaxation techniques such as deep breathing, imagery, head rolling, shoulder shrugging, rotating and stretching of the neck, leg lifts, and heel lifts with feet flat on the floor can effectively reduce tension while causing little or no distraction to those around you. It is recommended that you practice one or two of these techniques intermittently to avoid becoming tense. The more anxious and tense you become, the longer it will take you to relax.

A client who is to be admitted for minor surgery has a chest radiograph as part of the presurgical physical. The nurse is notified that the radiograph reveals that the client has pulmonary tuberculosis. What evidence of tuberculosis is provided by the radiograph? 1 Cavities caused by caseation 2 Sensitized T cells 3 Presence of acid-fast bacilli 4 Microscopic primary infection

4. Cavities caused by caseation Cavities are evident on radiograph. Necrotic lung tissue may liquefy, leaving a cavity (cavitation), or granulose tissue can surround the lesion, become fibrous, and form a collagenous scar around the tubercle (Ghon tubercle). Sensitized T cells are determined by a positive reaction to a tuberculin skin test, not on radiograph; a skin test only determines the presence of antibodies; it does not confirm active disease. Presence of acid-fast bacilli may be determined by a sputum culture, not by radiograph. Microscopic primary infection may be so small it does not appear on a radiograph.

A client with a history of emphysema is admitted with a diagnosis of acute respiratory failure with respiratory acidosis. Oxygen is being administered at 3 L/min nasal cannula. Four hours after admission, the client has increased restlessness and confusion followed by a decreased respiratory rate and lethargy. What should the nurse do? 1 Question the client about the confusion. 2 Change the method of oxygen delivery. 3 Percuss and vibrate the client's chest wall. 4 Discontinue or decrease the oxygen flow rate.

4. Discontinue or decrease the oxygen flow rate. With emphysema, it is believed that the respiratory center no longer responds to elevated carbon dioxide as the stimulus to breathe but rather to lowered oxygen levels; therefore, the oxygen being delivered must be lowered to supply enough for oxygenation without being so elevated that it negates the stimulus to breathe. However, the results of one recent study of clients with stable chronic obstructive pulmonary disease (COPD) indicate that the hypercarbic drive is preserved. More research is needed before this theory is applied clinically. A confused client cannot answer questions about the confusion. There are no indications that respiratory secretions have increased.

Which chest examination findings can be observed in a client with pneumonia? 1 Absent sounds on auscultation 2 Hyperresonance on percussion 3 Prolonged expiration on inspection 4 Increased fremitus over affected area on palpation

4. Increased fremitus over affected area on palpation Palpation in clients with pneumonia reveals increased fremitus over the affected area. Clients with pneumonia may have bronchial sounds initially and crackles, rhonchi, egophony later. Clients with atelectasis, however, may have absent sounds on auscultation. Hyperresonance on percussion may be observed in clients with asthma exacerbation or chronic obstructive pulmonary disease. Percussion in clients with pneumonia, however, may reveal dull sounds over the affected areas. Prolonged expiration on inspection is observed in clients with asthma exacerbations. However, clients with pneumonia may have tachypnea.

A client with a sucking chest wound has a large, tight dressing over the site. Which purpose of the dressing does the nurse consider when planning care for this client? 1 Protects the lung 2 Seals off major vessels 3 Prevents additional contamination of the wound 4 Maintains the appropriate pressure within the chest cavity

4. Maintains the appropriate pressure within the chest cavity A pressure dressing maintains negative intrathoracic pressure by limiting the amount of air rushing in from the outside; without an occlusive dressing this wound can result in a total pneumothorax and mediastinal shift, severely impeding respiration. The dressing does not prevent injury to the pleura. Major blood vessels are not occluded by a thoracic pressure dressing. It does not provide protection from additional contamination; a sterile dressing without pressure will prevent contamination. STUDY TIP: Rest is essential to the body and brain for good performance; think of it as recharging the battery. A run-down battery provides only substandard performance. For most students, it is better to spend 7 hours sleeping and 3 hours studying than to cut sleep to 6 hours and study 4 hours. The improvement in the rested mind's efficiency will balance out the difference in the time spent studying. Knowing your natural body rhythms is necessary when it comes to determining the amount of sleep needed for personal learning efficiency.

The nurse is providing postoperative care for an obese adult who had major abdominal surgery. The client has a history of smoking three packs of cigarettes daily. Which lab/diagnostic finding will the nurse check for the most accurate measurement of the client's respiratory status? 1 PaO2 2 PaCO2 3 Hemoglobin 4 Oxygen saturation

4. Oxygen saturation Oxygen saturation is a measure of the relationship between oxygen and hemoglobin; it measures the amount of oxygen available to tissues and provides a pulmonary assessment for clients at risk for hypoxia; pulse oximetry provides a continuous, noninvasive measurement of an individual's oxygen saturation. PaO2 is a measure of diffusion across the alveolar membrane. This value may decrease progressively in a heavy smoker; thus, it is not the most accurate measure of this client's postoperative respiratory status. Also, it requires an arterial puncture. PaCO2 is an accurate measure of alveolar ventilation and is elevated with hypoventilation. This may occur in any client after abdominal surgery because deep breathing often is painful. Also, an arterial puncture is necessary to acquire a specimen for testing. Hemoglobin is a measure of the blood's capacity to transport oxygen.

The son of a 65-year-old client said, "My father is suffering from chronic lung disease. He wakes suddenly from sleep and is unable to breathe." What condition does the nurse suspect in the client? 1 Orthopnea 2 Hemoptysis 3 Histoplasmosis 4 Paroxysmal nocturnal dyspnea

4. Paroxysmal nocturnal dyspnea The symptoms of paroxysmal nocturnal dyspnea (PND), which includes waking up suddenly with an inability to breathe, usually develop after the client has been lying down for several hours. PND often occurs in clients with chronic lung disease. Orthopnea is a condition that causes shortness of breath when lying down; this condition is relieved by sitting up. Hemoptysis is characterized by the presence of blood in the sputum; this condition is generally seen in clients with chronic bronchitis or lung cancer. Histoplasmosis is a fungal respiratory infection caused by the inhalation of contaminated dust.

A client with a history of emphysema develops a respiratory infection and is admitted to the hospital in acute respiratory distress. The client's arterial blood studies indicate pH 7.30, PO2 60 mm Hg, PCO2 55 mm Hg, and HCO3 23 mEq/L (23 mmol/L). How should the nurse interpret these findings? 1 Hypocapnia 2 Hyperkalemia 3 Generalized anemia 4 Respiratory acidosis

4. Respiratory acidosis The client is experiencing respiratory acidosis. The pH is less than the norm of 7.35 to 7.45, indicating acidosis [1] [2] [3]. The PO2 is less than the norm of 80 to 100 mm Hg. The PCO2 is increased more than the norm of 35 to 45 mm Hg. The HCO3 is within the norm of 21 to 28 mEq/L (21 to 28 mmol/L). These results indicate a respiratory etiology. The client's carbon dioxide level is increased (hypercapnia), not decreased. These values are unrelated to hyperkalemia; a serum potassium level of more than 5 mEq/L (5 mmol/L) indicates hyperkalemia. These values are unrelated to anemia; decreased levels of red blood cells (RBCs), hemoglobin, and hematocrit are related to anemia.

What order would the nurse follow for the assessment of the pharynx in a client with a respiratory disorder?

1. Inspect the mouth. 2. Observe the rise and fall of the soft palate. 3. Assess the symmetry of enlarged tonsils, if present. 4. Inspect the neck symmetry. 5. Palpate the lymph nodes. The assessment of the pharynx begins with an inspection of the mouth. By using a tongue depressor, the posterior pharynx and the rise and fall of the soft palate are observed. The next step is to assess for edema or ulceration and the symmetry of the enlarged tonsils. The neck is inspected for symmetry, alignment, masses, swelling, bruises, and the use of accessory neck muscles in breathing. The last step is to palpate the lymph nodes for size, shape, mobility with palpation, consistency, and tenderness.

The nurse should refer a client to the pulmonary clinic for suspected tuberculosis based on which clinical indicators reported during the initial client interview? Select all that apply. 1 Vomiting 2 Weight gain 3 Hemoptysis 4 Night sweats 5 Bilateral crackles

3,4 Erosion of lung tissue causes blood in the sputum, a classic sign of tuberculosis. Increased body temperature causes profuse diaphoresis, a classic sign of tuberculosis. Vomiting is associated with a gastrointestinal (GI) obstruction or cancer. Weight loss, not weight gain, is a sign of tuberculosis. Bilateral crackles are associated with excess fluid volume.

A nurse is caring for a client who has a tracheostomy tube and is on a ventilator. What must the nurse ensure about the tracheostomy tube? 1 Has an inner cannula 2 Is changed every week 3 Is cleansed once a day 4 Has a low-pressure cuff

4. Has a low-pressure cuff A low-pressure cuff permits tidal volume to reach the lungs while preventing tracheal necrosis. The tracheostomy tube can be a single-lumen tube or can have inner and outer cannulas. A tracheostomy tube does not have to be changed weekly. The tracheostomy should be cleaned every 8 hours and whenever necessary.

A client is admitted to the hospital with a diagnosis of laryngeal cancer. What is a common early sign of laryngeal cancer for which the nurse should assess in this client? 1 Aphasia 2 Dyspnea 3 Dysphagia 4 Hoarseness

Hoarseness Hoarseness is caused by the inability of the vocal cords to move adequately during speech when a tumor exists. Aphasia refers to an expressive or receptive communication deficit as a result of cerebral disease; it is not related to laryngeal cancer. Dyspnea is a late, not early, adaptation that occurs with laryngeal cancer when a tumor is large enough to obstruct air flow. Dysphagia is a late, not early, adaptation that occurs when the tumor is large enough to compress the esophagus.

A client who is homeless is hospitalized for alcohol withdrawal. When considering the type of personal protective equipment that is needed for the client's care, what condition does the nurse recall that homeless persons are at risk for? 1 Prostatitis 2 Tuberculosis 3 Osteoarthritis 4 Diverticulosis

Tuberculosis Medically underserved clients such as the homeless, clients who are alcohol or drug dependent, and those who have human immunodeficiency virus (HIV) infections are at risk for developing tuberculosis. Being homeless does not increase a person's risk for developing prostatitis, osteoarthritis, or diverticulosis. Test-Taking Tip: Multiple-choice questions can be challenging, because students think that they will recognize the right answer when they see it or that the right answer will somehow stand out from the other choices. This is a dangerous misconception. The more carefully the question is constructed, the more each of the choices will seem like the correct response.

A healthcare provider prescribes oropharyngeal suctioning as needed for a client in a coma. Which assessment made by the nurse indicates the need for suctioning? 1 Gurgling sounds with each breath 2 Fine crackles at the base of the lungs 3 Cyanosis in the nail beds of the fingers 4 Dry cough at increasingly frequent intervals

1. Gurgling sounds with each breath Secretions in the upper airway produce gurgling sounds that interfere with the free flow of air with each breath. Oropharyngeal suction will not address fine crackles at the base of the lungs. Cyanosis can result from a variety of problems unrelated to the presence of secretions; suctioning should be done only when secretions are blocking the airway. For a dry cough, suctioning is not needed in the absence of accumulated oropharyngeal secretions.

A client has a laryngectomy. The avoidance of which activity identified by the client indicates that the nurse's teaching about activities and the stoma is understood? 1 Water sports 2 Strenuous exercises 3 Sleeping with pillows 4 High-humidity environment

1. Water sports Water sports pose a severe threat; should water enter the stoma, the client will drown. Strenuous exercises are not harmful; as long as there is no obstruction, adequate oxygen will be available because the respiratory rate will increase. Pillows are not contraindicated, although care should be taken not to occlude the airway by any bedding while asleep. Humidity is desirable and helpful in keeping secretions liquefied.

A nurse is caring for a client with a history of chronic obstructive pulmonary disease (COPD) who develops a pneumothorax and has a chest tube inserted. Which primary purpose of the chest tube will the nurse consider when planning care? 1 Lessens the client's chest discomfort 2 Restores negative pressure in the pleural space 3 Drains accumulated fluid from the pleural cavity 4 Prevents subcutaneous emphysema in the chest wall

2. Restores negative pressure in the pleural space Negative pressure is exerted by gravity drainage or by suction through the closed system. Though the discomfort may be lessened as a result of the insertion of the chest tube, this is not the primary purpose. There is an accumulation of air, not fluid, when a pneumothorax occurs in a client with COPD. Subcutaneous emphysema in the chest wall is associated most commonly with clients receiving air under pressure, such as that received from a ventilator; subcutaneous emphysema can also occur with a chest tube.

The X-ray report of a client indicates a reduction in the alveolar surface area. Which condition can be inferred from this finding? 1 Acinus 2 Atelectasis 3 Hemoptysis 4 Histoplasmosis

2. atelectasis Atelectasis, a condition that involves alveolar collapse, may occur due to the absence of surfactants. This condition causes a reduction in the alveolar surface area, which in turn reduces the gas exchange. The structural unit consisting of a respiratory bronchiole, an alveolar duct, and an alveolar sac is known as an acinus. Histoplasmosis is a fungal disease caused by the inhalation of contaminated dust. Hemoptysis is blood in the sputum.

The nurse obtains a laboratory report that shows acid-fast rods in a client's sputum. Which disorder should the nurse consider may be related to these results? 1 Influenza virus 2 Diphtheria bacillus 3 Bordetella pertussis 4 Mycobacterium tuberculosis

4. Mycobacterium tuberculosis Mycobacterium tuberculosis [1] [2] is the acid-fast causative organism of tuberculosis. Acid-fast rods are not related to influenza viruses. The diphtheria bacillus is not an acid-fast rod. The microorganism that causes pertussis is not an acid-fast rod.

A client has been admitted for an upper respiratory tract infection secondary to chronic obstructive pulmonary disease (COPD). The nurse should expect which findings when auscultating the client's breath sounds? 1 Coarse crackles 2 Prolonged inspiration 3 Short, rapid inspiration 4 Normal breath sounds

1. Coarse crackles Coarse crackles and rhonchi most often are auscultated in COPD clients who have had an exacerbation. Clients will exhibit prolonged expiration, not prolonged inspiration. The client would not exhibit short, rapid inspiration or normal breath sounds with COPD. Test-Taking Tip: Identifying content and what is being asked about that content are critical to your choosing the correct response. Be alert for words in the stem of the item that are the same or similar in nature to those in one or two of the options. Example: If the item relates to and identifies stroke rehabilitation as its focus, and if only one of the options contains the word stroke in relation to rehabilitation, you are safe in identifying this choice as the correct response.

A client with a history of closed-angle glaucoma is scheduled for abdominal surgery. Because the client is extremely anxious, surgery is to be performed under general anesthesia. What should the nurse teach the client to do to prevent respiratory complications postoperatively? 1 Deep breathing techniques 2 Performing productive coughing 3 Turning from side to side frequently 4 Pant breathing while gently closing the eyelids

1. Deep breathing techniques Deep breathing is an intervention to prevent respiratory complications that does not increase intraocular pressure. Coughing is contraindicated because it increases intraocular pressure. Although turning from side to side is permitted, it is not as effective as deep breathing in preventing respiratory complications. Pant breathing is shallow breathing and will not prevent respiratory complications.

A client develops acute respiratory distress, and a tracheostomy is performed. Which intervention is most important for the nurse to implement when caring for this client? 1 Encouraging a fluid intake of 3 L daily 2 Suctioning via the tracheostomy every hour 3 Applying an occlusive dressing over the surgical site 4 Using cotton balls to cleanse the stoma with peroxide

1. Encouraging a fluid intake of 3 L daily Increased fluids help to liquefy secretions, enabling the client to clear the respiratory tract by coughing. Suctioning frequently will irritate the mucosal lining of the respiratory tract, which can result in more secretions. An occlusive dressing will block air exchange; the tracheostomy is now the client's airway. The use of cotton balls around a tracheostomy introduces the risk of aspiration of one of the cotton fibers; gauze should be used.

A community health nurse is educating a client who is interested in discontinuing cigarette smoking. What should the teaching plan include? 1 Helping the client set a date to stop smoking 2 Referring the client to the American Red Cross (Canada: Canadian Red Cross) 3 Encouraging the client to eat when the desire to smoke occurs 4 Telephoning the client several weeks after the preset target date

1. Helping the client set a date to stop smoking Setting a realistic target date to stop smoking [1] [2] can be motivating because it provides time to gather personal resources while committing to a specific timeframe. The American Heart Association (Canada: Heart and Stroke Foundation of Canada) and the American Lung Association (Canada: Canadian Lung Association) are appropriate agencies for referral, not the American Red Cross. Increasing eating may result in a weight gain that can precipitate reestablishing the habit of smoking to return to the former weight. The client should be called every three to five days, not weeks, after the target date for optimum support. Test-Taking Tip: Start by reading each of the answer options carefully. Usually at least one of them will be clearly wrong. Eliminate this one from consideration. Now you have reduced the number of response choices by one and improved the odds. Continue to analyze the options. If you can eliminate one more choice in a four-option question, you have reduced the odds to 50/50. While you are eliminating the wrong choices, recall often occurs. One of the options may serve as a trigger that causes you to remember what a few seconds ago had seemed completely forgotten.

Which statement appropriately describes tidal volume? 1. It is the volume of air inhaled and exhaled with each breath. 2. It is the amount of air remaining in the lungs after forced expiration. 3. It is the additional air that can be forcefully inhaled after normal inhalation. 4. It is the additional air that can be forcefully exhaled after normal exhalation.

1. It is the volume of air inhaled and exhaled with each breath Tidal volume is the volume of air inhaled and exhaled with each breath. Residual volume is the amount of air remaining in the lungs after forced expiration. Inspiratory reserve volume is the additional air that can be forcefully inhaled after normal inhalation. Expiratory reserve volume is the additional air that can be forcefully exhaled after normal exhalation.

A client who is a pipe smoker is diagnosed with cancer of the tongue. A hemiglossectomy and right radical neck dissection are performed. To ensure airway patency while the client is in the postanesthesia care unit, what should the nurse do? 1 Suction as needed 2 Apply an ice collar 3 Maintain a high-Fowler position 4 Encourage expectoration of secretions

1. Suction as needed After a hemiglossectomy a client will have difficulty swallowing and expectorating oral secretions because of the trauma of surgery. Although the application of an ice collar may limit edema or pain, it will not maintain patency of an airway that is compromised by secretions. A side-lying position will facilitate better drainage from the mouth. The client may not be reactive or have energy to cough or expectorate; the priority is to prevent secretions from entering the respiratory tract.

The registered nurse is teaching a group of nursing students about the characteristics of the five percussion notes. Which statements made by a student nurse indicate effective learning? Select all that apply. 1 "Resonance indicates the presence of trapped air." 2 "Dullness can be percussed over a consolidated lung." 3 "Hyperresonance is characteristic of normal lung tissue." 4 "Tympanic notes over the lung usually indicate a large pneumothorax." 5 "Flatness percussed over the lung fields indicates massive pleural effusion."

2,4,5 Dullness can be percussed over an atelectatic lung or a consolidated lung. Tympanic notes over the lung usually indicate a large pneumothorax. Flatness percussed over the lung fields indicates massive pleural effusion. These statements made by the student nurse indicate effective learning. Resonance is characteristic of normal lung tissue. Hyperresonance indicates the presence of trapped air.

During the evening after a paracentesis, the nurse identifies that the client, although denying any discomfort, is very anxious. Which action is best for the nurse to take? 1 Offer the client a back rub 2 Administer the prescribed opioid 3 Reinforce the primary healthcare provider's explanation of the procedure 4 Explore the client's concerns while administering the prescribed anxiolytic

4. Explore the client's concerns while administering the prescribed anxiolytic Sharing and discussing concerns often release anxieties; giving the prescribed anxiolytic will produce relaxation. Offering the client a back rub might relax the client but will do little to reduce the client's level of anxiety. The client is not in pain at this time but needs to share concerns. The procedure is over; this might be appropriate before the paracentesis.

What is the major factor a nurse considers when anticipating the method of oxygen administration to be prescribed by the healthcare provider for a client? 1 Activity level 2 Facial anatomy 3 Mental capacity 4 Pathologic condition

4. Pathologic condition Several modes are used for administration of oxygen; selection is based on the disease and the client's status. Although the other factors such as activity level, facial anatomy, and mental capacity will be taken into consideration, the ultimate decision is based on the pathologic condition and therapeutic needs.

The nurse provides teaching to a client who will begin to receive tube feedings after a total laryngectomy. The nurse concludes that the teaching was understood when the client makes which statement about tube feedings? 1 "I will need tube feedings until healing of the incision is complete." 2 "I will need tube feedings until the gag reflex returns." 3 "I will need tube feedings until the ability to belch is restored." 4 "I will need tube feedings until my oral feedings can be digested."

1. "I will need tube feedings until healing of the incision is complete." Food should be avoided until the area is healed completely; this will keep the area from becoming irritated and contaminated. Because of the alterations in structure, the gag reflex is no longer present. The ability to belch has no bearing on the decision to resume oral feedings. The ability to tolerate oral feedings is not lost; such feedings are withheld to prevent irritation to the surgical site until healing has taken place. Test-Taking Tip: Be alert for details. Details provided in the stem of the item, such as behavioral changes or clinical changes (or both) within a certain time period, can provide a clue to the most appropriate response or, in some cases, responses.

The nurse is caring for different clients in a healthcare setting who are diagnosed with respiratory disorders. Which client may have the anteroposterior chest diameter equal to the lateral chest and the slope of the ribs more horizontal to the spine? 1 Client 1: CF 2 Client 2: Bronchiectasis 3 Client 3: metabolic acidosis 4 Client 4: pulmonary edema

1. CF Client 1 with cystic fibrosis may have increased anteroposterior diameter; that is, the anteroposterior chest diameter is equal to the lateral chest measurement and the slope of the ribs are more horizontal to the spine. Client 2 with bronchiectasis may have finger clubbing with increased depth, bulk, and sponginess of the distal portion of the finger. Client 3 with metabolic acidosis may have Kussmaul respirations that are regular, rapid, and deep. Client 4 with pulmonary edema may sit in a tripod position, which involves leaning forward with arms and elbows supported on an overbed table.

After a lateral crushing chest injury, obvious right-sided paradoxical motion of a client's chest demonstrates multiple rib fractures, resulting in a flail chest. Which complication associated with this injury should the nurse assess in this client? 1 Mediastinal shift 2 Tracheal laceration 3 Open pneumothorax 4 Pericardial tamponade

1. Mediastinal shift Mediastinal structures move toward the uninjured lung, reducing oxygenation and venous return. Tracheal laceration is unlikely with a crushing injury to the chest. Flail chest is a closed chest injury; open pneumothorax results from a penetrating injury to the chest wall. Pericardial tamponade is associated with a cardiac contusion and usually occurs from a sternal, not lateral, compression injury.

A client who sustained trauma to the chest as a result of an injury has chest tubes inserted and is attached to a closed chest drainage system. When caring for this client, what should the nurse do? 1 Palpate the area around the tubes for crepitus 2 Clamp the chest tubes when suctioning the client 3 Empty the drainage chamber at the end of the shift 4 Change the client's dressing daily using aseptic technique

1. Palpate the area around the tubes for crepitus Leakage of air into the subcutaneous tissue is evidenced by a crackling sound when the area is palpated gently; this is referred to as crepitus. Hemostats should be readily available for any client with chest tubes in the event of a break in the drainage system; otherwise, clamping the tube is not necessary. The system is kept closed to prevent the pressure of the atmosphere from causing a pneumothorax; drainage levels are marked on the drainage chamber to measure output. To minimize the risk of a pneumothorax, the dressing is not changed routinely.

The nurse is teaching the client with chronic obstructive pulmonary disease (COPD) to use pursed-lip breathing (PLB). What is the rationale for the nurse's teaching? 1 Prolonged exhalation to decrease air trapping 2 Shortened inhalation to reduce bronchial swelling 3 Increased respiratory rate to improve arterial oxygenation 4 Decreased use of diaphragm to increase amount of inspired air

1. Prolonged exhalation to decrease air trapping Pursed-lip breathing works to decrease dyspnea and the respiratory rate through prolonging exhalation and prevention of alveolar collapse. PLB does not increase the length of inhalation and does not increase the respiratory rate. Use of the diaphragm occurs with diaphragmatic, or abdominal, breathing. STUDY TIP: Record the information you find to be most difficult to remember on 3" × 5" cards and carry them with you in your pocket or purse. When you are waiting in traffic or for an appointment, just pull out the cards and review again. This "found" time may add points to your test scores that you have lost in the past.

A nurse observes a client with acute bronchitis and emphysema sitting up in bed, appearing anxious and dyspneic. What should the nurse do first? 1 Provide oxygen at 2 L per minute 2 Encourage deep breathing and coughing 3 Administer the prescribed sedative and encourage rest 4 Suggest breathing into a paper bag for several minutes

1. Provide oxygen at 2 L per minute Low concentrations of oxygen will provide relief, do not reduce the stimulus to breathe, and prevent carbon dioxide narcosis. Respiratory obstruction causes difficulty on expiration; deep breathing may aggravate this problem. Pursed-lip breathing is advised to prevent the collapse of alveoli and promote the exhalation of carbon dioxide. Sedatives will depress further respirations and increase the carbon dioxide level. Chronic hypercapnia is present; additional carbon dioxide from breathing into a paper bag will add to the problem.

A firefighter is admitted to the emergency department with severe dermal and inhalation burns. On assessment, a nurse identifies tachycardia, tachypnea, and dyspnea. What term should the nurse document in the medical record when the following is heard on auscultation of the lungs of this client? 1 Stridor 2 Rhonchi 3 Crackles 4 Wheezes

1. Stridor Stridor is an adventitious breath sound. It is a high-pitched, shrill, harsh sound generally occurring on inspiration, but it can occur on expiration. Depending on its severity, it can be heard on auscultation of the lung or with the naked ear. Inhalation burns can cause a heat injury of the upper respiratory tract (nasopharynx, oropharynx, and larynx). Edema in response to an upper respiratory tract heat injury can produce airway obstruction, especially the epiglottitis, resulting in stridor. Rhonchi (gurgles) are adventitious breath sounds. They are continuous, low-pitched, coarse sounds often described as having a snoring or moaning quality. Rhonchi indicate partial bronchial obstruction caused by mucus or other fluids in the airway, bronchial hyperreactivity, or the presence of a tumor. Crackles (rales) are adventitious breath sounds. Crackles are fine, short, interrupted crackling sounds best heard on inspiration but may be heard on expiration. They are heard on auscultation of the base of the lung as air passes over retained secretions within the alveoli. A wheeze is an adventitious breath sound. It is a continuous, high-pitched, squeaky, musical sound best heard on exhalation. It is commonly heard over all lung fields and can be auscultated or heard by the naked ear. Wheezes are caused by narrowing of the lumen of the respiratory passages; they are associated with asthma, bronchitis, croup syndromes, lung infections, pulmonary edema, emphysema, or other chronic obstructive lung conditions.

Which action performed by the nursing student during the chest examination of a client needs correction? 1 Placing the stethoscope over bony prominences 2 Palpating two ribs inferiorly in the midaxillary line 3 Dividing the anterior and posterior lungs into thirds 4 Listening to at least one cycle of inspiration and expiration

1. placing the stethoscope over bony prominences The stethoscope should be placed over the lung tissue and not over bony prominences during chest auscultation. The nursing student should palpate the two ribs inferiorly in the midaxillary line and around the posterior chest. When documenting the location of lung sounds, the nursing student should divide the anterior and posterior lungs into thirds to describe the sounds. At each placement of the stethoscope, the nursing student should listen to at least one cycle of inspiration and expiration.

A nurse uses abdominal-thoracic thrusts (Heimlich maneuver) when an older adult in a senior center chokes on a piece of meat. Which volume of air is the basis for the efficacy of the abdominal thrusts to expel a foreign object in the larynx? 1 Tidal 2 Residual 3 Vital capacity 4 Inspiratory reserve

2. Residual The residual volume is the amount of air remaining in the lungs after maximum exhalation. Usually tidal, vital capacity, and inspiratory reserve volumes are under the individual's control. The force exerted by the abdominal thrust [1] [2] surpasses that which the individual is voluntarily capable of exerting.

A client has a laryngectomy and radical neck dissection for cancer of the larynx. Two tubes from the area of the incision are connected to portable wound drainage systems. Inspection of the neck reveals moderate edema even though the drainage systems are functioning. Which clinical indicator should the nurse assess in the client? 1 Crackles 2 Restlessness 3 Loss of the gag reflex 4 Cloudy wound drainage

2. Restlessness The client has a high risk for airway obstruction from the edema; restlessness and dyspnea indicate cerebral hypoxia. Crackles come from the alveoli, part of the lower airway; the surgery involves the upper airway. There is no evidence of abdominal distention. Loss of the gag reflex is unimportant. The pharyngeal opening is sutured closed, and a tracheal stoma is formed; the trachea is anatomically separate from the esophagus. Cloudy drainage may indicate infection; however, this is not an immediate postoperative complication.

A nurse auscultates a client's lungs and hears a fine crackling sound in the left lower lung during respiration. The nurse charts, "crackles and rhonchi in the left lower lung." What does this documentation represent? 1 A nursing diagnosis 2 An inaccurate interpretation 3 A correct nursing assessment 4 An accurate conclusion if crepitus was ruled out

2. an inaccurate interpretation Rhonchi are coarse sounds heard over the larger airways; including rhonchi in the record makes the documentation inaccurate. Crackles and rhonchi are clinical indicators, not a nursing diagnosis. It is incorrect to use the term rhonchi to refer to crackling sounds in the lower lung. Crepitus, which indicates subcutaneous emphysema, is unrelated to auscultated breath sounds. Test-Taking Tip: Practicing a few relaxation techniques may prove helpful on the day of an examination. Relaxation techniques such as deep breathing, imagery, head rolling, shoulder shrugging, rotating and stretching of the neck, leg lifts, and heel lifts with feet flat on the floor can effectively reduce tension while causing little or no distraction to those around you. It is recommended that you practice one or two of these techniques intermittently to avoid becoming tense. The more anxious and tense you become, the longer it will take you to relax.

A client is diagnosed with cancer of the larynx and is scheduled for a total laryngectomy. What should the nurse initially include in the preoperative teaching plan? 1 Have a speech therapist visit 2 Give a detailed explanation of the surgery 3 Allow the client an opportunity to ask questions 4 Explain breathing exercises to be performed postoperatively

3. Allow the client an opportunity to ask questions Allowing opportunities for questions is the initial intervention. Before teaching, the nurse must determine the client's areas of concern; knowledge generally reduces anxiety, and learning increases when anxiety decreases. Having a speech therapist visit is not beneficial for all preoperative clients scheduled for a laryngectomy. Although a detailed explanation of the surgery may be part of the information included in preoperative teaching, it is provided by the primary healthcare provider, and the teaching should begin at the client's level. The client will be dependent on a respirator postoperatively; breathing exercises are not necessary immediately after surgery.

client is admitted to the intensive care unit with a diagnosis of acute respiratory distress syndrome. Which clinical finding should the nurse expect when assessing this client? 1 Hypertension 2 Tenacious sputum 3 Altered mental status 4 Slowed rate of breathing

3. Altered mental status Altered mental status is secondary to cerebral hypoxia, which accompanies acute respiratory distress syndrome (ARDS); cognition and level of consciousness are reduced. Hypotension occurs because of cardiac hypoxia. The sputum is not tenacious, but it may be frothy if pulmonary edema is present. Breathing is fast and shallow. Test-Taking Tip: A psychological technique used to boost your test-taking confidence is to look into a mirror whenever you pass one and say out loud, "I know the material, and I'll do well on the test." Try it; many students have found that it works because it reduces "test anxiety."

A client develops respiratory alkalosis. When the nurse is reviewing the laboratory results, which finding is consistent with respiratory alkalosis? 1 An elevated pH, elevated PCO2 2 A decreased pH, elevated PCO2 3 An elevated pH, decreased PCO2 4 A decreased pH, decreased PCO2

3. An elevated pH, decreased PCO2 In respiratory alkalosis the pH level is elevated because of loss of hydrogen ions; the PCO2 level is low because carbon dioxide is lost through hyperventilation. An elevated pH, elevated PCO2 is partially compensated metabolic alkalosis. A decreased pH, elevated PCO2 is respiratory acidosis. A decreased pH, decreased PCO2 is metabolic acidosis with some compensation.

A nurse administers oxygen at 2 L/min via nasal cannula to a client with chronic obstructive pulmonary disease (COPD). By administering a low concentration of oxygen to this client, the nurse is preventing which physiologic response? 1 Decrease in red cell formation 2 Rupture of emphysematous bullae 3 Depression in the respiratory center 4 Excessive drying of the respiratory mucosa

3. Depression in the respiratory center It is believed that clients with COPD should be given low concentrations of oxygen because a decreased oxygen blood level is the stimulus for breathing for these clients. However, the results of a recent study of clients with stable COPD indicate that the hypercarbic drive is preserved with oxygen concentrations higher than 2 L/min. More research is needed before this theory is applied clinically. Prolonged hypoxia stimulates erythrocyte production; the goal of therapy is to relieve hypoxia. The pressure, rather than the concentration, at which oxygen is administered increases the risk of rupture of emphysematous bullae. The concentration of oxygen is unrelated to its humidification. To prevent its drying effects on secretions and the mucosa, oxygen should be humidified.

The nurse is caring for a client with emphysema. During assessment, the nurse expects to auscultate which type of breath sounds? 1 Pleural friction rub 2 Crackles and gurgles 3 Diminished breath sounds 4 Expiratory wheeze and cough

3. Diminished breath sounds Breath sounds will be decreased in clients with emphysema because of reduced airflow, pleural effusion, or lung parenchymal destruction. A pleural friction rub occurs when one layer of the pleural membrane slides over the other during breathing; this is associated with pleurisy. Crackles indicate fluid in the alveoli, which is associated with heart failure or infection; rhonchi signify airway obstruction, not emphysema. Expiratory wheezing and coughing are associated with asthma or bronchitis. Test-Taking Tip: Avoid taking a wild guess at an answer. However, should you feel insecure about a question, eliminate the alternatives that you believe are definitely incorrect, and reread the information given to make sure you understand the intent of the question. This approach increases your chances of randomly selecting the correct answer or getting a clearer understanding of what is being asked. Although there is no penalty for guessing, the subsequent question will be based, to an extent, on the response you give to the question at hand; that is, if you answer a question incorrectly, the computer will adapt the next question accordingly based on your knowledge and skill performance on the examination up to that point.

Besides providing reassurance, what should nursing interventions for a client who is hyperventilating be focused on? 1 Administering oxygen 2 Using an incentive spirometer 3 Having the client breathe into a paper bag 4 Administering an IV containing bicarbonate ions

3. Having the client breathe into a paper bag Reassurance decreases anxiety and slows respirations; the bag is used so that exhaled carbon dioxide can be rebreathed to resolve respiratory alkalosis and return the client to an acid-base balance. Administering oxygen is not necessary because there is no evidence of hypoxia. Using an incentive spirometer is used to prevent atelectasis. The client is already alkalotic; bicarbonate ions will increase the problem.

What response provides evidence that a client with chronic obstructive pulmonary disease (COPD) understands the nurse's instructions about an appropriate breathing technique? 1 Inhales through the mouth 2 Increases the respiratory rate 3 Holds each breath for a second at the end of inspiration 4 Progressively increases the length of the inspiratory phase

3. Holds each breath for a second at the end of inspiration Holding each breath for a second at the end of inspiration allows added time for gas exchange at alveolar capillary beds. Inhalation should occur through the nose to moisten, filter, and warm the air. Increasing the respiratory rate does not allow for prolonged expirations, thus decreasing the effectiveness of respirations. The expiratory phase should be lengthened, and exhalation should occur through pursed lips to prevent alveolar collapse. Test-Taking Tip: You have at least a 25% chance of selecting the correct response in multiple-choice items. If you are uncertain about a question, eliminate the choices that you believe are wrong and then call on your knowledge, skills, and abilities to choose from the remaining responses.

A nurse is teaching a community health class about the risk factors for cancer. Which factor has the least influence in predisposing an individual to cancer of the larynx? 1 Air pollution 2 Heavy alcohol ingestion 3 Inadequate dental hygiene 4 Chronic respiratory infection

3. Inadequate dental hygiene Inadequate dental hygiene may predispose a person to oral infections but is involved only remotely in laryngeal neoplasms because of the anatomic relationship of the oral cavity and the larynx. Irritation by air pollutants may initiate tissue changes that can lead to malignancy. Alcohol is an irritant that may initiate tissue changes that result in a malignant neoplasm. Tissue alterations caused by repeated microbiologic stress may result in a malignant neoplasm.

After abdominal surgery, a goal is to have the client achieve alveolar expansion. The nurse determines that this goal is most effectively achieved by what method? 1 Postural drainage 2 Pursed-lip breathing 3 Incentive spirometry 4 Sustained exhalation

3. Incentive spirometry Incentive spirometry expands collapsed alveoli and enhances surfactant activity, thereby preventing atelectasis. Postural drainage helps clear accumulated secretions from the pulmonary tree; it does not directly promote alveolar expansion. Pursed-lip breathing promotes sustained exhalation, not inhalation. Sustained exhalation promotes the collapse, not expansion, of alveoli.

A client arrives at a walk-in clinic complaining of chest pain on inspiration and shortness of breath, and has a temperature of 102° F (38.9° C). What term should the nurse document in the medical record when the following is heard on auscultation of the lateral lung fields of this client? 1 Stridor 2 Rhonchi 3 Pleural friction rub 4 High-pitched crackles

3. Pleural friction rub A pleural friction rub is a rough, scratching, grating, creaking sound caused by inflamed pleural surfaces rubbing together. It is frequently associated with chest pain. It is most often heard at the end of inspiration and on the beginning of expiration, when the lateral and anterior lung fields are auscultated. A pleural friction rub is associated with pleurisy, tuberculosis, pulmonary emboli, pneumonia, and lung cancer. Stridor is an adventitious breath sound. It is a high-pitched, shrill, harsh sound generally occurring on inspiration, but it can occur on expiration. Depending on its severity, it can be heard on auscultation of the lung or with the naked ear. Stridor occurs with laryngeal obstruction (upper airway obstruction) and is associated with croup syndromes. Rhonchi (gurgles) are adventitious breath sounds. They are continuous, low-pitched, coarse sounds often described as having a snoring or moaning quality. Rhonchi indicate partial bronchial obstruction caused by mucus or other fluids in the airway, bronchial hyperreactivity, or the presence of a tumor. High-pitched crackles (rales) are adventitious breath sounds. Crackles are fine, short, interrupted crackling sounds best heard on inspiration, but may be heard on expiration. They are heard on auscultation of the base of the lung as air passes over retained secretions within the alveoli.

After abdominal surgery a client should be encouraged to turn from side to side and to engage in deep breathing exercises. The nurse explains that these activities are essential to prevent which condition? 1 Metabolic acidosis 2 Metabolic alkalosis 3 Respiratory acidosis 4 Respiratory alkalosis

3. Respiratory acidosis Shallow respirations, bronchial tree obstruction, and atelectasis compromise gas exchange in the lungs; an elevated carbon dioxide level leads to acidosis [1] [2]. Metabolic acidosis is caused by a loss of bicarbonate from the lower gastrointestinal tract, which is associated with diarrhea. Metabolic alkalosis is caused by excessive loss of hydrogen ions from gastric decompression or excessive vomiting. Respiratory alkalosis is caused by increased expiration of carbon dioxide, a component of carbonic acid.

A nurse identifies 12 mm of induration at the site of a Mantoux test when a client returns to the health office to have it read. Which explanation of this result should the nurse give to the client? 1 Result is negative, and follow-up is not needed. 2 The disease is active, and medication is required. 3 Additional tests are needed, such as a chest x-ray. 4 Outcome is inconclusive, and the test will be repeated in six weeks.

3. additional tests are needed, such as a chest x-ray Induration of 10 mm or more is a positive result indicative of being infected with Mycobacterium tuberculosis organism; additional tests, including chest x-ray and sputum cultures, are now warranted to identify whether active disease is present. Test result is positive, not negative; further testing is necessary. A positive test indicates only exposure to and possible infection with tuberculosis. Waiting to do further testing is unsafe.

A client who experienced smoke inhalation has a negative chest x-ray and arterial blood gases that demonstrate PaO2 of 75 mm Hg, PaCO2 of 45 mm Hg, and pH of 7.35. Which intervention should the nurse anticipate will be prescribed by the healthcare provider? 1 Deep suctioning 2 Bronchodilators 3 Breathing exercises 4 Mechanical ventilation

3. breathing exercise Breathing exercises are needed. The client has hypoxemia; the expected range for PaO2 is 80 to 100 mm Hg. This intervention expands the alveoli, moves secretions toward the mouth to be expectorated, and increases the amount of oxygen that is delivered to alveolar capillary beds. Routine suctioning may injure already traumatized tissues and is contraindicated. Bronchodilators and mechanical ventilation are not indicated at this time based upon the x-ray results and PaCO2 and pH results.

A nurse is caring for a client with severe dyspnea who is receiving oxygen via a Venturi mask. What should the nurse do when caring for this client? 1 Assess frequently for nasal drying. 2 Keep the mask tight against the face. 3 Monitor oxygen saturation levels when eating. 4 Set the oxygen flow at the highest setting possible.

3. monitor O2 level when eating Because the mask cannot be worn when eating, the client may become hypoxic. A nasal cannula may be needed to deliver oxygen while the client is eating. Nasal drying usually is not a problem with the use of a Venturi mask. Nasal drying occurs more frequently when a nasal cannula is used. Too tight a fit for the mask is uncomfortable and may cause damage to the skin. The mask should fit snugly but not be too tight. The oxygen should be set at the level prescribed by the healthcare provider. Test-Taking Tip: Prepare for exams when and where you are most alert and able to concentrate. If you are most alert at night, study at night. If you are most alert at 2 am, study in the early morning hours. Study where you can focus your attention and avoid distractions. This may be in the library or in a quiet corner of your home. The key point is to keep on doing what is working for you. If you are distracted or falling asleep, you may want to change when and where you are studying.

During the assessment of a client who was admitted to the hospital because of a productive cough, fever, and chills, the nurse percusses an area of dullness over the right posterior lower lobe of the lung. Which medical diagnosis will the nurse most likely observe documented in the client's electronic records? 1 Pleurisy 2 Bronchitis 3 Pneumonia 4 Emphysema

3. pneumonia The data presented indicate an infectious process within the lung. The classic clinical findings associated with pneumonia are a productive cough (sputum is purulent, blood-tinged, or rust-colored), fever, chills, pleuritic chest discomfort, and dyspnea. Percussion is dulled over areas of consolidation. The cardinal clinical findings associated with pleurisy are pain in the lower lobe at the height of inspiration and a pleural friction rub. Although fever and chills can occur later in bronchitis, the cardinal clinical findings associated with bronchitis are irritating productive cough, chest pain, and shortness of breath. The cardinal clinical findings associated with emphysema are barrel chest, resonance on percussion, and air trapping. Test-Taking Tip: Avoid choosing answers that use words such as always, never, must, all, and none. If you are confused about the question, read the choices, label them true or false, and choose the answer that is the odd one out (i.e., the one false one or the one true one). When a question is framed in the negative, such as "When assessing for pain, you should not," the false option is the correct choice.

A client has chronic obstructive pulmonary disease (COPD). To decrease the risk of CO2 intoxication (CO2 narcosis), what should the nurse do? 1 Initiate pulmonary hygiene to clear air passages of trapped mucus 2 Instruct to deep breathe slowly with inhalation longer than exhalation 3 Encourage continuous rapid panting to promote respiratory exchange 4 Administer oxygen at a low concentration to maintain respiratory drive

4. Administer oxygen at a low concentration to maintain respiratory drive With chronically high levels of carbon dioxide it is believed that decreased oxygen levels become the stimulus to breathe; high oxygen administration negates this mechanism. Initiating pulmonary hygiene to clear air passages of trapped mucus is an appropriate intervention, but is not directly related to CO2 intoxication (CO2 narcosis). Encouraging continuous rapid panting to promote respiratory exchange will not bring oxygen into the alveoli for exchange; nor will it adequately remove carbon dioxide because it will increase bronchiolar obstruction. Inhalation should be of regular depth, and expiration should be prolonged to prevent carbon dioxide trapping (air trapping). Test-Taking Tip: Come to your test prep with a positive attitude about yourself, your nursing knowledge, and your test-taking abilities. A positive attitude is achieved through self-confidence gained by effective study. This means (a) answering questions (assessment), (b) organizing study time (planning), (c) reading and further study (implementation), and (d) answering questions (evaluation).

Endotracheal intubation and positive-pressure ventilation are instituted because of a client's deteriorating respiratory status. Which is the priority nursing intervention? 1 Facilitate verbal communication 2 Prepare the client for emergency surgery 3 Maintain sterility of the ventilation system 4 Assess the client's response to the mechanical ventilation

4. Assess the client's response to the mechanical ventilation The effectiveness of therapy is measured by the client's response. Endotracheal intubation does not permit verbal communication. Preparing the client for emergency surgery is presumptive; the database is inadequate for this conclusion. Maintaining sterility of the ventilation system is important but not the priority; the client is a higher priority than equipment. STUDY TIP: Becoming a nursing student automatically increases stress levels because of the complexity of the information to be learned and applied and because of new constraints on time. One way to decrease stress associated with school is to become very organized so that assignment deadlines or tests do not come as sudden surprises. By following a consistent plan for studying and completing assignments, you can stay on top of requirements and thereby prevent added stress.

The nurse assists with a client's yearly physical examination. After the examination is completed, the client is diagnosed with tuberculosis. Which action best reflects appropriate epidemiological follow-up? 1 Obtaining a list of people the client has had contact with over the past year 2 Suggesting that the client notify acquaintances that the disease has developed 3 Requiring employees at the client's work site to have chest x-rays as soon as possible 4 Encouraging close family members, friends, and coworkers of the client to have a skin test

4. Encouraging close family members, friends, and coworkers of the client to have a skin test People exposed to the client need screening, and it is appropriate to start with a skin test; those with positive reactions should be tested further. Making a list of all the people the client has had contact with over the past year is not necessary and is impractical; only recent close contacts need to be identified. Only close friends and contacts need to be notified. Only employees in proximity to the client need be screened, and a chest x-ray film is not the best initial screening method.

A nurse is providing education to a coworker who is caring for a client who is scheduled to have a thoracentesis for a pleural effusion. Which information will be appropriate for the nurse to include? 1 The thoracentesis procedure uses the principle of positive pressure. 2 It is common for a sclerosing agent to be instilled at the end of the procedure. 3 Clients will have temporary increased dyspnea immediately after the procedure. 4 Rapid removal of large amounts of fluid may precipitate cardiovascular collapse.

4. Rapid removal of large amounts of fluid may precipitate cardiovascular collapse. The mechanism is unclear, but cardiovascular collapse probably is caused by fluid shifts. A thoracentesis uses the principle of negative pressure. Use of a sclerosing agent is not commonly done. Dyspnea should be relieved immediately; if dyspnea increases, a pneumothorax should be suspected.

What should the nurse expect when assessing a client with pleural effusion? 1 Crackles or rhonchi at the posterior of the lungs 2 Deviation of the trachea toward the affected side 3 Increased resonance on percussion of the affected area 4 Reduced or absent breath sounds at the base of the lung

4. Reduced or absent breath sounds at the base of the lung Compression of the lung by fluid that accumulates at its base reduces expansion and air exchange. Crackles or rhonchi at the posterior of the lungs are not associated with pleural effusion. If tracheal deviation occurs, it is away from the affected side. Dullness is produced on percussion of the affected area. Test-Taking Tip: Avoid looking for an answer pattern or code. There may be times when four or five consecutive questions have the same letter or number for the correct answer.

A client is admitted with possible tuberculosis. To make a definitive diagnosis, the nurse expects which diagnostic test to be prescribed? 1 Chest x-ray film 2 Tuberculin skin test 3 Pulmonary function test 4 Sputum test for acid-fast bacilli

4. Sputum test for acid-fast bacilli When the tubercle bacilli are stained with an acid, they turn red and are not decolorized by an acid-alcohol wash; they are acid fast. The rods are visible upon microscopic examination. Chest x-ray film reflects pulmonary status but does not identify the organism if a lesion is found. Tuberculin skin test indicates the presence of antibodies but is not diagnostic of the disease; it just means the client has been exposed. Pulmonary function test reflects pulmonary status but does not identify the organism if a lesion is found.

A client is hospitalized with a diagnosis of emphysema. The nurse provides teaching and should begin with which aspect of care? 1 The disease process and breathing exercises 2 How to control or prevent respiratory infections 3 Using aerosol therapy, especially nebulizers 4 Priorities in carrying out everyday activities

The disease process and breathing exercises Clients need to understand the disease process and how interventions, such as breathing exercises, can improve ventilation. Learning to control or prevent respiratory infections is important, but it should be taught later. Although it is helpful to know about aerosol therapy and nebulizers, knowing how to use aerosol therapy, especially nebulizers, should be taught later. Although it is important to teach the client how to set priorities in carrying out everyday activities, this should be taught later. Test-Taking Tip: The most reliable way to ensure that you select the correct response to a multiple-choice question is to recall it. Depend on your learning and memory to furnish the answer to the question. To do this, read the stem, and then stop! Do not look at the response options yet. Try to recall what you know and, based on this, what you would give as the answer. After you have taken a few seconds to do this, then look at all of the choices and select the one that most nearly matches the answer you recalled. It is important that you consider all the choices and not just choose the first option that seems to fit the answer you recall. Remember the distractors. The second choice may look okay, but the fourth choice may be worded in a way that makes it a slightly better choice. If you do not weigh all the choices, you are not maximizing your chances of correctly answering each question.

After surgery, a client is extubated in the postanesthesia care unit. Which clinical manifestations should the nurse expect if the client is experiencing acute respiratory distress? Select all that apply. 1 Confusion 2 Hypocapnia 3 Tachycardia 4 Constricted pupils 5 Slow respiratory rate

1, 3. Inadequate cerebral oxygenation produces restlessness and confusion. Tachycardia occurs as the body attempts to compensate for the lack of oxygen. A low carbon dioxide level in the blood (hypocapnia) occurs with an increase in respiratory rate. The pupils dilate, not constrict, with hypoxia. An elevated respiratory rate (tachypnea), not a slow respiratory rate (bradypnea), occurs.

A client with chronic obstructive pulmonary disease (COPD) reports chest congestion, especially upon awakening in the morning. What should the nurse suggest to the client? 1 Use a humidifier in the bedroom. 2 Sleep with two or more pillows. 3 Cough regularly even if the cough does not produce sputum. 4 Cough and deep breathe each night before going to sleep.

1, use a humidifier in the bedroom A humidifier will help liquefy secretions and promote their expectoration. Sleeping on pillows facilitates breathing; it does not relieve chest congestion. Nonproductive coughing should be avoided because it is irritating and exhausting. Deep breathing and coughing at night will not help relieve early-morning congestion.

What clinical indicators should the nurse expect to identify when assessing an individual with a spontaneous pneumothorax? Select all that apply. 1 Hematemesis 2 Shortness of breath 3 Unilateral chest pain 4 Increased thoracic motion 5 Mediastinal shift toward the involved side

2,3. With the reduction of surface area for gaseous exchange, the client experiences shortness of breath, tachycardia, and rapid, shallow respirations. Sudden chest pain occurs on the affected side; it may also involve the arm and shoulder. Bloody vomitus is unrelated to pneumothorax. Decreased chest motion occurs because of failure to inflate the involved lung. The shift toward the unaffected side results from pressure with the pneumothorax.

A client has a history of falling while playing football and now reports pain in the nose and difficulty breathing. What condition may the client have? 1 Crepitus 2 Sinusitis 3 Fracture of the nose 4 Upper respiratory tract infection

3, fracture of the nose Fractures of the nose often result from injuries received during falls, sports activities, car crashes, or physical assaults. Nose fractures may lead to difficulty in breathing. Crepitus is crackling of the skin on palpitation. Sinusitis is an inflammation of the tissues lining the sinuses. In an upper respiratory tract infection, a stuffy nose and itching results in difficulty breathing. However, pain may not be present.

The nurse in the postanesthesia care unit is caring for a client who had a left-sided pneumonectomy. Which goal is priority? 1 Replace blood loss 2 Maintain ventilatory exchange 3 Maintain closed chest drainage 4 Replace supplemental oxygenation

3, maintain ventilatory exchange Oxygen and carbon dioxide exchange is essential for life and is the priority. Blood replacement is not the priority. Closed chest drainage is unnecessary with a left-sided pneumonectomy because there is no lung to reinflate. Supplemental oxygenation is not the priority. STUDY TIP: Laughter is a great stress reliever. Watching a short program that makes you laugh, reading something funny, or sharing humor with friends helps decrease stress.

When providing discharge teaching for a young female client who had a pneumothorax, it is important that the nurse include the signs and symptoms of a recurring pneumothorax. What is the most important symptom that the nurse should teach the client to report to the healthcare provider? 1 Substernal chest pain 2 Episodes of palpitation 3 Severe shortness of breath 4 Dizziness when standing up

3. Severe shortness of breath Severe shortness of breath may indicate a recurrence of the pneumothorax because one lung is unable to meet the oxygen demands of the body. A pneumothorax causes sharp chest pain on the involved side, not substernally. Usually palpitations reflect a cardiac, not a respiratory problem. Dizziness when standing up is not specific to a pneumothorax; this is orthostatic hypotension, which may be related to a variety of medical problems. Test-Taking Tip: Stay away from other nervous students before the test. Stop reviewing at least 30 minutes before the test. Take a walk, go to the library and read a magazine, listen to music, or do something else that is relaxing. Go to the test room a few minutes before class time so that you are not rushed in settling down in your seat. Tune out what others are saying. Crowd tension is contagious, so stay away from it.

A nurse instructs a client to breathe deeply to open collapsed alveoli. What should the nurse include in the explanation of the relationship between alveoli and improved oxygenation? 1 "The alveoli need oxygen to live." 2 "The alveoli have no direct effect on oxygenation." 3 "Collapsed alveoli increase oxygen demands." 4 "Oxygen is exchanged for carbon dioxide in the alveolar membrane."

4 "Oxygen is exchanged for carbon dioxide in the alveolar membrane." The exchange of oxygen and carbon dioxide occurs in the alveolar membrane. Therefore, if the alveoli collapse, this exchange cannot occur. Explaining this process in simple terms to a client may increase compliance with recommended breathing exercises aimed at improving oxygenation. Alveoli do have a direct effect on oxygenation. The statements that alveoli need oxygen to live and that collapsed alveoli increase oxygen demands are nonspecific regarding the pathophysiology of the alveolar membrane. STUDY TIP: Avoid planning other activities that will add stress to your life between now and the time you take the licensure examination. Enough will happen spontaneously; do not plan to add to it.

A nurse is teaching a client with a diagnosis of pulmonary tuberculosis about recovery after discharge. What is the most important intervention for the nurse to include in this plan? 1 Ensuring sufficient rest 2 Changing lifestyle routines 3 Breathing clean outdoor air 4 Taking medications as prescribed

4. Taking medications as prescribed Tubercle bacilli are particularly resistant to treatment and can remain dormant for long periods. Drugs must be taken consistently, or more drug-resistant forms may recolonize and flourish. Although a balance between activity and rest is desirable, it is not the priority. A change in lifestyle is not necessary. Although clean, fresh air is desirable, it is not the priority. STUDY TIP: Regular exercise, even if only a 10-minute brisk walk each day, aids in reducing stress. Although you may have been able to enjoy regular sessions at the health club or at an exercise class several times a week, you now may have to cut down on that time without giving up a set schedule for an exercise routine. Using an exercise bicycle that has a book rack on it at home, the YMCA, or a health club can help you accomplish two goals at once. You can exercise while beginning a reading assignment or while studying notes for an exam. Listening to lecture recordings while doing floor exercises is another option. At least a couple of times a week, however, the exercise routine should be done without the mental connection to school; time for the mind to unwind is necessary, too.

A serious train accident occurs in the community. At the scene of the accident, a triage nurse is identifying and labeling victims according to triage acuity principles. With what color tag should the nurse label a client who is experiencing respiratory distress? 1 Red 2 Black 3 Green 4 Yellow

Red A red tag (priority I) indicates a client with respiratory distress, trauma or bleeding, or neurological deficits that need immediate treatment. Victims who are deceased are labeled with a black tag. These individuals are transported to a temporary morgue after clients who have a chance for survival are attended to. A green tag (priority III) indicates a client who needs care that can wait for hours. Although clients with sprains, rashes, and minor pain can wait hours for treatment, they need to be reassessed every 1 to 2 hours to ensure that their condition did not deterioraterm-7te. If their condition deteriorates, they should be relabeled according to their level of need. A yellow tag (priority II) indicates injuries that need treatment within 2 hours. Although people who have sustained simple fractures, lacerations, or fevers can wait for treatment for 2 hours, they need to be reassessed every 30 minutes to ensure that their condition did not deteriorate. If their condition deteriorates, they should be relabeled with a red tag (priority I), indicating the need for immediate treatment. Test-Taking Tip: Read every word of each question and option before responding to the item. Glossing over the questions just to get through the examination quickly can cause you to misread or misinterpret the real intent of the question.


Ensembles d'études connexes

Human Resources Management Section 2 MGT 364

View Set

Evaluating Science in the Media: Evolution / Primate Phylogenetics Researchers Swinging from Tree to Tree/ Connecting the Concepts: Chordate Clades

View Set

Marketing, Grewal & Levy, 7th Edition, chapter 12

View Set

Idioms with apple and bananas. Glossary by Naves https://quizlet.com/_97lq8t

View Set